Đến nội dung

Stranger411

Stranger411

Đăng ký: 24-04-2012
Offline Đăng nhập: 15-09-2015 - 12:18
***--

#397889 Những người phát cuồng vì tramyvodoi

Gửi bởi Stranger411 trong 17-02-2013 - 23:58

Các cháu trên faceboook đang phát cuồng :D
Mong bạn tramyvodoi đừng giận nhá :))
Hình đã gửi

Và đây là hậu quả của những cháu đua đòi :-ss
Hình đã gửi


#392825 Đếm bằng $v_p$

Gửi bởi Stranger411 trong 03-02-2013 - 15:47

Cho bảng hình chữ nhật kích thước $m$ x $n$ ($m,n \in \mathbb{Z^+}$) và các số nguyên dương $p \le m$ và $q \le n$. Người ta điền các số $1,2,3, ..., m$ x $n$ vào các ô trong bảng. Một số tự nhiên trong bảng được gọi là "xấu" nếu nó nhỏ hơn $p$ số cùng cột và $q$ số cùng hàng. Đếm số cách xếp sao cho các số xấu có trong bảng là nhỏ nhất và hãy chỉ ra 1 cách xếp như vậy.



Ps: Đang viết 1 bài về $v_p$ và đây là một ứng dụng trong tổ hợp của nó.
  • LNH yêu thích


#392819 Bất khả quy trên $\mathbb{Z[x]}$

Gửi bởi Stranger411 trong 03-02-2013 - 15:29

Cho đa thức $P(x)= \prod_{i=1}^{n}( x- a_i)$ với $n \ge 5$ và các $a_i \in \mathbb{Z}$ phân biệt. Chứng minh nếu tam thức $ a x^2 + bx +1$ bất khả quy trên $\mathbb{Z[x]}$ thì đa thức $ a P(x)^2 + b P(x) +1$ cũng bất khả quy trên $\mathbb{Z[x]}$


#392817 $c \le \frac{1}{4n}$

Gửi bởi Stranger411 trong 03-02-2013 - 15:17

Cho dãy số $a_1, a_2, ..., a_n$ và số $c$ thỏa mãn 2 điều kiện:
1) $a_n =0$
2) $a_k = c+ \sum_{i=k}^{n-1}(a_i + a_{i+1})$ $\forall k=\overline{0,n-1}$.
Chứng minh $c \le \frac{1}{4n}$


#387048 $P(x^2 -2002) \vdots P(x)$

Gửi bởi Stranger411 trong 15-01-2013 - 22:00

Có tồn tại hay không đa thức P(x) thỏa:
(1) $degP(x) = 2003$
(2) $P(x^2 -2002) \vdots P(x)$ ($\forall x \in \mathbb{R}$)


ps: hồi chiều làm kiểm tra bị bể bài này :((((


#385774 VMO 2013 - Bài 1. Hệ phương trình

Gửi bởi Stranger411 trong 11-01-2013 - 23:00

Cách khác: (không dùng Minkowski :D)
ĐKXĐ: $x\neq \dfrac{m\pi }{2}$, $y\neq \dfrac{n\pi }{2}$ ($m$, $n\in \mathbb{Z}$), $xy\geq 0$, $x+y\neq 0$.
Ta chứng minh:
$(\sqrt{{{\sin }^{2}}x+\frac{1}{{{\sin }^{2}}x}}+\sqrt{{{\cos }^{2}}y+\frac{1}{{{\cos }^{2}}y}})^2 + (\sqrt{{{\sin }^{2}}y+\frac{1}{{{\sin }^{2}}y}}+\sqrt{{{\cos }^{2}}x+\frac{1}{{{\cos }^{2}}x}})^2 \ge 20$

Áp dụng $\frac{1}{\sin ^2 x} + \frac{1}{\cos ^2 x} \ge 4$ $ \forall x \neq 0$, ta có:
$(\sqrt{{{\sin }^{2}}x+\frac{1}{{{\sin }^{2}}x}}+\sqrt{{{\cos }^{2}}y+\frac{1}{{{\cos }^{2}}y}})^2 + (\sqrt{{{\sin }^{2}}y+\frac{1}{{{\sin }^{2}}y}}+\sqrt{{{\cos }^{2}}x+\frac{1}{{{\cos }^{2}}x}})^2 $
$\ge 10 + 2 \sqrt{{{\sin }^{2}}x+\frac{1}{{{\sin }^{2}}x}}.\sqrt{{{\cos }^{2}}y+\frac{1}{{{\cos }^{2}}y}} + 2 \sqrt{{{\sin }^{2}}y+\frac{1}{{{\sin }^{2}}y}}.\sqrt{{{\cos }^{2}}x+\frac{1}{{{\cos }^{2}}x}}$

Vậy chỉ cần chứng minh:
$ \sqrt{{{\sin }^{2}}x+\frac{1}{{{\sin }^{2}}x}}.\sqrt{{{\cos }^{2}}y+\frac{1}{{{\cos }^{2}}y}} + \sqrt{{{\sin }^{2}}y+\frac{1}{{{\sin }^{2}}y}}.\sqrt{{{\cos }^{2}}x+\frac{1}{{{\cos }^{2}}x}} \ge 5$ $ \forall x \neq 0$
từ đó quay về chứng minh:
$ \left(\sin^2x + \dfrac{1}{\sin^2x}\right)\left(\cos^2x + \dfrac{1}{\cos^2x}\right) \geq \left(\dfrac{5}{2}\right)^2.$và $ \left(\sin^2y + \dfrac{1}{\sin^2y}\right)\left(\cos^2y + \dfrac{1}{\cos^2y}\right) \geq \left(\dfrac{5}{2}\right)^2.$

Sử dụng bất đẳng thức Cauchy-Schwarz và AM-GM ta có
$$ \begin{aligned} \left(\sin^2x + \dfrac{1}{\sin^2x}\right)\left(\cos^2x + \dfrac{1}{\cos^2x}\right) \ & \geq \left(|\sin x\cos x|+\dfrac{1}{|\sin x\cos x|}\right)^2 \\ & =\left(\dfrac{|\sin 2x|}{2}+\dfrac{1}{2|\sin 2x|}+\dfrac{3}{2|\sin 2x|}\right)^2 \\ & \geq \left( 1+\dfrac{3}{2}\right)^2=\left(\dfrac{5}{2}\right)^2.\end{aligned}$$
Ta có (đpcm).
Suy ra: $\tan x = \tan y= \pm 1$ và $x=y$
Từ đó, ta được $ x=y= \dfrac{\pi }{4}+\dfrac{k\pi }{2}$ ($k\in \mathbb{Z}$).


#381518 $p \in A$

Gửi bởi Stranger411 trong 29-12-2012 - 11:15

Cho tập $A=\left \{ x|x=a^2+2b^2 , a,b\in \mathbb{Z},b \neq 0\right \}$ và 1 số nguyên tố $p$.
Chứng minh rằng nếu $p^2 \in A$ thì $p \in A$


#375898 cho n là số nguyên dương lớn hơn 1. CMR $2^n-1$ không chia hết cho n

Gửi bởi Stranger411 trong 07-12-2012 - 22:20

cho n là số nguyên dương lớn hơn 1. CMR $2^n-1$ không chia hết cho n

Ý tưởng ko khác gì mấy vs nguyenta98, chủ yếu là xét $v_2$ thôi.
Giải:
Xét phân tích tiêu chuẩn của $n=\prod\limits_{i = 1}^h {{p_i}^{{k_i}}}$ với các số nguyên tố ${p_1} < {p_2} < ... < {p_h}$. Trong đó ${p_i} = 1 + {2^{{r_i}}}{m_i}$ ($m_i$ lẻ) $\Rightarrow n \equiv 1(\bmod {m})$
Đặt $n - 1 = {2^m}t$ $\Rightarrow {2^{{2^m}t}} \equiv - 1(\bmod {p_i})$
Mà $- 1 \equiv {2^{{2^m}t{m_i}}} \equiv {2^{({p_i} - 1)t}} \equiv 1(\bmod {p_i})$
Vậy $p_i = 2$ (Vô lí)
Q.E.D

Remark: Chắc nguyenta98 có biết bài này:
Tìm tất cả số tự nhiên $n$ sao cho: $n|{2^n} + 2$


#375486 $\frac{n}{m}=\sum_{k=1}^{p-...

Gửi bởi Stranger411 trong 06-12-2012 - 00:19

Cho $p>3$ là một số nguyên tố.

Đặt $\begin{cases}{n\over m}=\sum_{k=1}^{p-1}{1\over k}\\ \mathrm{gcd}(n,m)=1\end{cases}$

Chứng minh rằng: $p\big| n$

Bài này có nhiều cách giải lắm thầy ạ :D
Theo em cách giải ngắn gọn nhất là sử dụng $\mathbb{Z}/p\mathbb{Z}$

Lời giải:
Ta xét bài toán trong $\mathbb{Z}/p\mathbb{Z}$, do tính tồn tại duy nhất của nghich đảo modulo, ta có:
$\overline{1}^{-1} + \overline{2}^{-1} +...+ \overline{p-1}^{-1} = \overline{1} + \overline{2} +...+ \overline{p-1} = \overline{1+2+...+ p-1} = \overline{\frac{p(p-1)}{2}} \equiv 0$
Q.E.D


#375471 $C_{{p^r}}^p \equiv {p^{r - 1}...

Gửi bởi Stranger411 trong 05-12-2012 - 23:06

CMR: $C_{{p^r}}^p \equiv {p^{r - 1}}(\bmod {p^r})$

Đây là một bài toán không quá khó để phải dùng đến các kiến thức về số ${v_p}$ như vậy.

Lời giải:
Áp dụng công thức $kC_{n}^k =nC_{k-1}^{n-1}$, ta được:
$$C_{p^r}^p - p^{r-1}= p^{r-1} \left ( C_{p^r-1}^{p-1} -1 \right )$$
Từ đó, ta chỉ cần chứng minh:
$$C_{p^r-1}^{p-1} \equiv 1 (mod p) (*)$$
Nhưng đây chỉ là một hệ quả của định lí Lucas.

Remark: Ta có thể chứng minh một bổ đề khác mạnh hơn $(*)$ như sau:

Cho số nguyên tố $p$ và các số tự nhiên $k,a$, trong đó $0 \le a \le p^{k-1}$.
Chứng minh: $C_{p^{k-1}}^{a} \equiv (-1)^a ( mod p)$

Và bổ đề trên cũng chỉ là 1 hệ quả của định lí Lucas


#368417 $\binom{kn}{n}\vdots k^n$

Gửi bởi Stranger411 trong 10-11-2012 - 15:31

Tìm tất cả số nguyên $n>1$ sao cho:
a) $\binom{3n}{n}\vdots 3^n$
b) $\binom{kn}{n}\vdots k^n$ với mọi $k>1$.


ps: bài kiểm tra 1 tiết lớp mình :(
có rất ít người làm được cả 2 câu.



#368262 $\sum_{k=0}^{\left\lfloor\frac{n...

Gửi bởi Stranger411 trong 09-11-2012 - 21:48

Dạo này box tổ hợp và rời rạc của VMF có vẻ trầm lắng!
Để tránh tình trạng này kéo dài, tôi xin khuấy động bằng một bài nho nhỏ

Cho số nguyên $n\ge 3$. Chứng minh đẳng thức:

$\sum_{k=0}^{\left\lfloor\frac{n}{3}\right\rfloor} \binom{n}{3k} = \dfrac{2^n+(-1)^n\left(3\left\lfloor\frac{n}{3}\right\rfloor-3\left\lfloor\frac{n-1}{3}\right\rfloor-1\right)}{3}$

Chào các anh. Bài này dùng hàm sinh kết hợp với RUF tức là Root of Unity Filter đó ạ :D

Định lí: Cho số nguyên dương $n$. Đặt $k=cos \frac{2 \pi}{n}+i sin \frac{2 \pi}{n}$.
Xét đa thức $f(x)=a_n x^n+a_{n-1} x^{n-1}+...+a_1 x+a_0$. Khi đó:
$a_0 + a_{n} + a_{2n} + ... = \frac{1}{n}[f(1)+f(k)+f(k^2)+...+f(k^{n-1})]$

Bài toán: Đặt $k=cos \frac{2 \pi}{3}+i .sin \frac{2 \pi}{3}$.
Áp dụng định lí trên, ta được:
$\sum_{k=0}^{\left\lfloor\frac{n}{3}\right\rfloor} \binom{n}{3k} = \frac{(1+k)^n + (1+ k^2 )^n + (1+1)^n}{3}$
$=\frac{(-1)^n(k^n + k^{2n}) + 2^n}{3}$
từ đó, ta có kết quả bài toán.

Remark: Ngoài ra, ta còn kết quả khác:
Nếu $n$ chia hết cho 3: $\sum_{k=0}^{\infty } \binom{n}{3k} = \frac{2^n + 2(-1)^n}{3}$
Nếu $n$ không chia hết cho 3: $\sum_{k=0}^{\infty } \binom{n}{3k} = \frac{2^n + (-1)^{n+1}}{3} $


ps hxthanh@: thầy cho em hỏi ạ :D hồi rất lâu rồi ấy,thầy có lập 1 topic tên là "Dãy số đầu năm" trong box Đại số THCS. Sau này mod chuyển đi đâu mất :( Thầy cho em cái link để tham khảo được không ạ, Hồi đó em còn non nên chưa đọc được nhiều :D


#359606 Tìm nghiệm nguyên

Gửi bởi Stranger411 trong 06-10-2012 - 22:43

Tìm $p,q\in \mathbb{P}$ thỏa mãn $3pq\mid a^{3pq}-a$ với mọi $a\in \mathbb{Z}^+$

Nếu biết giới hạn $p,q$ và chọn $a$ là căn nguyên thủy của $n$ ngay từ đầu thì bài toán sẽ gọn hơn rất nhiều.
Giả sử $p \ge q$

+ Cho $a=3$, ta có:
${3^{pq}} \equiv 3\left( {\bmod 3pq} \right) \Rightarrow 3\left( {{3^{pq - 1}} - 1} \right) \vdots 3pq \Rightarrow p,q > 3$

+ Cho ${a^{\varphi \left( n \right)}} \equiv 1\left( {\bmod n} \right)$ ( $a$ là căn nguyên thủy của $n$)
Theo định lí Fermat nhỏ: ${a^{p - 1}} \equiv 1\left( {\bmod p} \right)$
Vì ${a^{3pq - 1}} \equiv 1\left( {\bmod p} \right) \Rightarrow p - 1|3pq - 1 \Rightarrow p - 1|3q - 1$
Chứng minh tương tự: $q - 1|3p - 1$
Vì $p \ge q$ nên $3q - 1 \in \left\{ {p - 1;2\left( {p - 1} \right);3\left( {p - 1} \right)} \right\}$
Thay vào điều kiện bài toán, ta được: $\boxed{(p,q)=(11,17),(17,11)}$


#353030 $p \equiv 1 (\bmod m)$

Gửi bởi Stranger411 trong 08-09-2012 - 23:40

Tổng quát hóa từ một bài toán:
Cho số nguyên tố $p \equiv 1 (\bmod m)$ với $m>2$. Chứng minh:
\[\prod\limits_{i = 1}^p {\left( {{i^{m - 1}} + {i^{m - 2}} + \ldots + i + 1} \right)} \equiv 0 (\bmod p)\]


Bài này dùng 1 tí kiến thức về hệ thu gọn.
Bên mathlink post mấy tháng rồi mà chưa có lời giải :-<


#347148 Tìm các số nguyên dương $a,b,c$ sao cho $\frac{a^{2}+b^{2...

Gửi bởi Stranger411 trong 16-08-2012 - 10:37

sai từ chỗ này và nguyên nhân là do làm tắt $p|{(2a + b)^2} + 3{b^2}$
$ \Rightarrow \left( {\frac{{ - 3}}{p}} \right) = 1$
muốn dùng lengdre(hay tiếng việ gọi là thặng dư toàn phương) trước tiên ta phải đưa nó về dạng (mà ở đây) là
a2$\equiv$-3 (mod p) cái đã,mà ở đây muốn đưa về dạng này ta phải giả sử a không chia hết cho p,''vậy nên thiếu TH a,b chia hết cho p'',mà TH này luôn đúng,nếu không thấy dc thì cho a=b=p ta có 12p2 chia hết cho p ,vì vậy có giải kiểu gì đi nữa vẫn phải thông qua a,b,c chia hết cho p rồi mới giải tiếp,nên không có cách bạn stranger nói

Nói chuyện vs Uyenha cực kì bực mình @@!
Mình và mọi người đã ko muốn nói rồi mà bạn cứ thích cãi cùn.

Trước đó, MOD đã gộp bớt vài bài của bạn để tránh spam trong topic.
Thắc mắc thì ko phải là tội nhưng cứ nói dai như thế người ta chả thích tí nào đâu bạn :)

Mời bạn tham khảo thêm về kí hiệu Lengdre:
File gửi kèm  Cong Thuc Legendre.pdf   67.83K   1641 Số lần tải